Proving the Existence of a Subset with Divisible Sum in Number Theory

Click For Summary
The discussion revolves around proving the existence of a subset of natural numbers whose sum is divisible by a given natural number n. The initial approach involved the pigeonhole principle, which led to confusion regarding congruency classes modulo n. Participants clarified that there are n congruency classes and explored the implications of sums of subsets of the original set. If one of the sums is congruent to zero, the proof is complete; if not, the existence of two sums in the same congruency class leads to a new sum that is congruent to zero modulo n. The conversation concludes with a successful resolution of the problem using these concepts.
AdrianZ
Messages
318
Reaction score
0

Homework Statement


Suppose n is a natural number and A is a subset of natural number with n elements. Prove that a subset of A exists that the sum of its elements is dividable by n.

The Attempt at a Solution


well, This problem is harder than I can solve it. I first tried to use the pigeonhole principle to prove the existence of such a subset of A. well, It wasn't easy to prove it that way as I thought. so, now I feel that I'm stuck and have no idea of how to solve it. Any ideas?
 
Physics news on Phys.org
You're on the right track with PHP. How many congruancy classes are there modulo n? Think about this for a bit, and I have another hint if you need one.
 
Robert1986 said:
You're on the right track with PHP. How many congruancy classes are there modulo n? Think about this for a bit, and I have another hint if you need one.

I've already thought about the number of congruency classes a lot. There are n congruency classes of modulo n ([0],...,[n-1]). In fact the Euclidean algorithm of division forces that. but the question is still so confusing. so another hint would be appreciated.
 
OK, let a_1,a_2,\ldots,a_n be the elements of your n-element set. Consider these sums:

a_1 \equiv m_1 mod n
a_1 + a_2 \equiv m_2 mod n
\vdots
a_1 + a_2 + \cdots + a_n \equiv m_n mod n

Now, what would it mean if one of the m_i's were 0? OK, now what if none of them are 0? How many congruancy classes does this leave? How many sums are there above? If a,b,c are integers and a \equiv c mod n and b \equiv c mod n what can you say about a-b mod n?
 
Robert1986 said:
OK, let a_1,a_2,\ldots,a_n be the elements of your n-element set. Consider these sums:

a_1 \equiv m_1 mod n
a_1 + a_2 \equiv m_2 mod n
\vdots
a_1 + a_2 + \cdots + a_n \equiv m_n mod n

OK, now what if none of them are 0? How many congruancy classes does this leave?How many sums are there above? If a,b,c are integers and a \equiv c mod n and b \equiv c mod n what can you say about a-b mod n?

well, if one of them is zero, we're done. if none of them is zero, then we'll have n-1 congruency classes and n such sums. that would mean that two of these sums should be in the same congruency class. therefore if we subtract those two from each other we'll have a new sum that is congruent to 0 modulo n and we're done.

Thanks for the help.
 
Beautiful!
 
Question: A clock's minute hand has length 4 and its hour hand has length 3. What is the distance between the tips at the moment when it is increasing most rapidly?(Putnam Exam Question) Answer: Making assumption that both the hands moves at constant angular velocities, the answer is ## \sqrt{7} .## But don't you think this assumption is somewhat doubtful and wrong?

Similar threads

Replies
1
Views
2K
Replies
20
Views
4K
  • · Replies 1 ·
Replies
1
Views
2K
  • · Replies 5 ·
Replies
5
Views
2K
  • · Replies 7 ·
Replies
7
Views
2K
  • · Replies 8 ·
Replies
8
Views
11K
  • · Replies 2 ·
Replies
2
Views
5K
  • · Replies 18 ·
Replies
18
Views
2K
Replies
2
Views
1K
  • · Replies 2 ·
Replies
2
Views
2K